Solutions détaillées de neuf exercices sur les suites numériques (fiche 01).
Cliquer ici pour accéder aux énoncés.

icone-math-OS-Exos
exercice 1 facile

Si la formule de récurrence qui définit la suite u ne comportait pas le terme -\frac{1}{2}, ce serait facile, car on aurait affaire à une suite géométrique de raison \frac{1}{3}.

Tâchons de nous ramener à cette situation.

Notons \lambda l’unique réel vérifiant \lambda=\frac{\lambda}{3}-\frac{1}{2}, autrement dit :

    \[\boxed{\lambda=-\frac{3}{4}}\]

En soustrayant membre à membre les égalités :

    \[ \left\{\begin{array}{ccc}u_{n+1} & = & {\displaystyle\frac{u_{n}}{3}-\frac{1}{2}}\\\\\lambda & = & {\displaystyle\frac{\lambda}{3}-\frac{1}{2}}\end{array}\right.\]

on obtient :

    \[ u_{n+1}-\lambda=\frac{1}{3}\left(u_{n}-\lambda\right)\]

Ceci prouve que la suite v définie par :

    \[ \forall n\in\mathbb{N},\thinspace v_{n}=u_{n}-\lambda=u_{n}+\frac{3}{4}\]


est géométrique de raison \frac{1}{3}, et donc que :

    \[ \forall n\in\mathbb{N},\thinspace v_{n}=\frac{v_{0}}{3^{n}}\]

Comme v_{0}=u_{0}+\frac{3}{4}=-\frac{1}{4}, il vient finalement :

    \[ \boxed{\forall n\in\mathbb{N},\thinspace u_{n}=-\frac{1}{4\times3^{n}}-\frac{3}{4}}\]

En particulier, la suite u converge vers -\frac{3}{4}.

exercice 2 facile

Par hypothèse, on a pour tout n\in\mathbb{N} :

    \[\left\{\begin{array}{ccc}a_{n+1} & = & \left(1-p\right)a_{n}+pb_{n}\\b_{n+1} & = & pa_{n}+\left(1-p\right)b_{n}\end{array}\right.\]

d’où en ajoutant membre à membre :

    \[a_{n+1}+b_{n+1}=a_{n}+b_{n}\qquad\left(\spadesuit\right)\]

et en soustrayant :

    \[a_{n+1}-b_{n+1}=\left(1-2p\right)\left(a_{n}-b_{n}\right)\qquad\left(\clubsuit\right)\]

D’après \left(\spadesuit\right) la suite a+b est constante. Donc :

    \[ \forall n\in\mathbb{N},\thinspace a_{n}+b_{n}=r+s\]

Et d’après \left(\clubsuit\right) la suite a-b est géométrique de raison 1-2p. Donc :

    \[ \forall n\in\mathbb{N},\thinspace a_{n}-b_{n}=\left(1-2p\right)^{n}\left(r-s\right)\]

Avec ces deux formules en poche, on calcule aisément a_{n} et b_{n} (en effectuant la demi-somme et la demi-différence) :

    \[ \boxed{\forall n\in\mathbb{N},\thinspace\left\{\begin{array}{ccc}a_{n} & = & \frac{1}{2}\left(r+s+\left(1-2p\right)^{n}\left(r-s\right)\right)\\\\b_{n} & = & \frac{1}{2}\left(r+s-\left(1-2p\right)^{n}\left(r-s\right)\right)\end{array}\right.}\]

On peut à présent conclure.

Comme p\in\left]0,1\right[, alors \left|1-2p\right|<1 et donc {\displaystyle \lim_{n\rightarrow\infty}\left(1-2p\right)^{n}=0.}

Ainsi, les suites a et b convergent, toutes les deux, vers \displaystyle{\frac{r+s}{2}}.

exercice 3 facile

Pour tout n\geqslant1 :

    \[ q_{n+1}=1+\prod_{i=0}^{n}q_{i}=1+q_{n}\prod_{i=0}^{n-1}q_{i}\]

et donc :

    \[ \boxed{q_{n+1}=1+q_{n}\left(q_{n}-1\right)}\]

relation qui est encore vraie pour n=0.

Si l’on note, pour tout n\in\mathbb{N}^{\star} :

    \[ S_{n}=\sum_{i=0}^{n-1}\frac{1}{q_{i}}\]

alors :

    \[S_{1}=\frac{1}{2},\qquad S_{2}=\frac{1}{2}+\frac{1}{3}=\frac{5}{6},\qquad S_{3}=\frac{1}{2}+\frac{1}{3}+\frac{1}{7}=\frac{41}{42}\]


    \[ S_{4}=\frac{1}{2}+\frac{1}{3}+\frac{1}{7}+\frac{1}{43}=\frac{1\thinspace805}{1\thinspace806}\]

On remarque qu’il s’agit de fractions de plus en plus voisines de 1. Plus précisément, il semble bien que, pour tout n\in\mathbb{N}^{\star} :

    \[\boxed{S_{n}=1-\frac{1}{q_{n}-1}}\]

Montrons ceci par récurrence. Comme on vient de le voir, cette propriété est vraie pour n=1; on peut donc passer directement à l’hérédité.

Supposons la formule vraie pour un certain n. On calcule :

    \[ S_{n+1}=S_{n}+\frac{1}{q_{n}}=1-\frac{1}{q_{n}-1}+\frac{1}{q_{n}}=1-\frac{1}{q_{n}\left(q_{n}-1\right)}\]

D’après la formule encadrée, on a donc :

    \[ S_{n+1}=1-\frac{1}{q_{n+1}-1}\]

comme souhaité.

Tout d’abord, cette suite est bien définie et à termes strictement positifs. Du coup, x_{n}>1 pour tout n\in\mathbb{N}^{\star}. En cas de convergence vers un réel \ell, un passage à la limite dans cette inégalité montre que \ell\geqslant1 (et, en particulier : \ell\neq0).

En passant à la limite dans la formule de récurrence, on voit aussi que :

    \[ \ell=1+\frac{1}{\ell}\]

donc :

    \[ \ell^{2}-\ell-1=0\]

ce qui prouve que :

    \[ \ell\in\left{ \frac{1-\sqrt{5}}{2},\:\frac{1+\sqrt{5}}{2}\right} \]

Mais la première de ces deux valeurs est exclue car négative. Bref : la convergence de la suite \left(x_{n}\right)_{n\in\mathbb{N}} n’est toujours pas établie, mais si elle a lieu, c’est nécessairement vers le nombre d’or, que nous notons désormais \gamma :

    \[\boxed{\gamma=\frac{1+\sqrt{5}}{2}}\]

Le principal intérêt de ce qui précède est de nous éclairer sur la valeur de la seule limite possible.

Prouvons maintenant la convergence de cette suite. Pour tout n\in\mathbb{N}^{\star} :

    \begin{eqnarray*}x_{n+1}-\gamma & = & \left(1+\frac{1}{x_{n}}\right)-\left(1+\frac{1}{\gamma}\right)\\& = & \frac{\gamma-x_{n}}{\gamma x_{n}}\end{eqnarray*}


donc, vu que x_{n}>1 :

    \[ \left|x_{n+1}-\gamma\right|\leqslant\frac{1}{\gamma}\left|x_{n}-\gamma\right|\]

d’où par une récurrence immédiate :

    \[ \forall n\in\mathbb{N}^{\star},\:\left|x_{n}-\gamma\right|\leqslant\frac{1}{\gamma^{n-1}}\left|x_{1}-\gamma\right|\]

Comme \gamma>1, cette dernière majoration permet de conclure que :

    \[ \lim_{n\rightarrow\infty}\left(x_{n}-\gamma\right)=0\]

Autrement dit :

    \[ \boxed{\lim_{n\rightarrow\infty}x_{n}=\frac{1+\sqrt{5}}{2}}\]

L’illustration ci-dessous montre le calcul des premiers termes :

Pour tout x>1 :

    \begin{eqnarray*}\frac{1}{x-1}-\frac{2}{x}+\frac{1}{x+1} & = & \frac{x\left(x+1\right)-2\left(x^{2}-1\right)+x\left(x-1\right)}{x\left(x^{2}-1\right)}\\& = & \frac{2}{x\left(x^{2}-1\right)}>0\end{eqnarray*}


d’où :

    \[ \frac{1}{x-1}+\frac{1}{x}+\frac{1}{x+1}>\frac{3}{x}\]

Il en résulte que, pour tout n\in\mathbb{N}^{\star} :

    \[ \sum_{k=1}^{n}\left(\frac{1}{3k-1}+\frac{1}{3k}+\frac{1}{3k+1}\right)>\sum_{k=1}^{n}\frac{1}{k}\]

c’est-à-dire (en ajoutant 1 à chaque membre) :

    \[\boxed{H_{3n+1}>1+H_{n}}\]

Si la suite \left(H_{n}\right)_{n\geqslant1} était convergente, sa limite L vérifierait L\geqslant1+L, ce qui est absurde. Cette suite est donc divergente et, comme elle est croissante (ce qui est évident : H_{n+1}-H_{n}=\frac{1}{n+1}>0 pour tout n\geqslant1), elle diverge vers +\infty.

On attribue cette méthode à Pietro Mengoli \left(1625-1686\right). Elle évoque une preuve plus classique, qui repose sur le fait que, pour tout n\in\mathbb{N}^{\star} :

    \[ H_{2n}>\frac{1}{2}+H_{n}\]

et qui est aussi plus ancienne : on la trouve chez Nicolas d’Oresme ( ca 1323 – 1382).

La relation

    \[ y_{k+1}=\left(k+1\right)y_{k}+1\]

peut s’écrire, après division par \left(k+1\right)! :

    \[ \frac{y_{k+1}}{\left(k+1\right)!}-\frac{y_{k}}{k!}=\frac{1}{\left(k+1\right)!}\]

On en déduit, après sommation (pour 0\leqslant k\leqslant n-1) :

    \[ \frac{y_{n}}{n!}-y_{0}=\sum_{k=0}^{n-1}\frac{1}{\left(k+1\right)!}\]

c’est-à-dire (vu que y_{0}=0 par hypothèse) :

    \[y_{n}=n!\sum_{k=1}^{n}\frac{1}{k!}\]

Or, il est bien connu que, pour tout t\in\mathbb{R} :

    \[ \lim_{n\rightarrow\infty}\sum_{k=0}^{n}\frac{t^{k}}{k!}=e^{t}\]

et donc, lorsque n\rightarrow\infty :

    \[ \boxed{y_{n}\sim\left(e-1\right)n!}\]

L’hypothèse :

    \[ \forall n\in\mathbb{N},\thinspace b_{n+1}-b_{n}\leqslant a_{n+1}-a_{n}\]


peut s’écrire :

    \[ \forall n\in\mathbb{N},\thinspace b_{n+1}-a_{n+1}\leqslant b_{n}-a_{n}\]

Elle exprime donc la décroissance de la suite b-a.

Comme la suite a est convergente, elle est nécessairement bornée. Et comme la suite b est bornée par hypothèse, on voit que b-a l’est aussi et elle est en particulier minorée.

On invoque alors le théorème de la limite monotone : la suite b-a est décroissante est minorée, donc convergente.

Pour finir, la suite b est convergente puisque c’est la somme de deux suites convergentes.

On passe au logarithme :

    \[ \ln\left(P_{n}\right)=\frac{1}{n}\sum_{k=1}^{n}\ln\left(n^{2}+k^{2}\right)\]


c’est-à-dire :

    \[ \ln\left(P_{n}\right)=\frac{1}{n}\sum_{k=1}^{n}\left[2\ln\left(n\right)+\ln\left(1+\frac{k^{2}}{n^{2}}\right)\right]\]


ou encore :

    \[ \ln\left(P_{n}\right)=2\ln\left(n\right)+R_{n}\]


où l’on a posé :

    \[ R_{n}=\frac{1}{n}\sum_{k=1}^{n}\ln\left(1+\frac{k^{2}}{n^{2}}\right)\]

On reconnaît une somme de Riemann attachée à l’application \left[0,1\right]\rightarrow\mathbb{R},\thinspace t\mapsto\ln\left(1+t^{2}\right). Par conséquent :

    \[ \lim_{n\rightarrow\infty}R_{n}=\int_{0}^{1}\ln\left(1+t^{2}\right)\thinspace dt\]

Si l’on intègre par parties en posant :

    \begin{eqnarray*}u'\left(t\right)=1 & ; & v\left(t\right)=\ln\left(1+t^{2}\right)\\u\left(t\right)=t & ; & v'\left(t\right)=\frac{2t}{1+t^{2}}\end{eqnarray*}


ce qui donne :

    \begin{eqnarray*}\int_{0}^{1}\ln\left(1+t^{2}\right)\thinspace dt & = & \left[t\ln\left(1+t^{2}\right)\right]_{0}^{1}-2\int_{0}^{1}\frac{t^{2}}{1+t^{2}}\thinspace dt\\& = & \ln\left(2\right)-2\int_{0}^{1}\left(1-\frac{1}{1+t^{2}}\right)\thinspace dt\\& = & \ln\left(2\right)-2\left(1-\frac{\pi}{4}\right)\end{eqnarray*}

Notons K cette valeur. On a montré, à ce stade que lorsque n\rightarrow\infty :

    \[ \ln\left(P_{n}\right)=2\ln\left(n\right)+K+o\left(1\right)\]

Il en résulte que :

    \[ P_{n}\sim e^{K}n^{2}\]


ou, de façon plus explicite :

    \[ \boxed{P_{n}\sim2\thinspace e^{\frac{\pi}{2}-2}n^{2}}\]

exercice 9 difficile

Vues les hypothèses :

    \[ \lim_{n\rightarrow\infty}\left[\left(e^{a_{n}}-1-a_{n}\right)+\left(e^{b_{n}}-1-b_{n}\right)+\left(e^{c_{n}}-1-c_{n}\right)\right]=0\]


Or, on sait (par convexité de l’exponentielle ou par une preuve directe) que :

    \[ \forall t\in\mathbb{R},\,e^{t}-1-t\geqslant0\]


Il en résulte que :

    \[\begin{array}{ccc}{\displaystyle \lim_{n\rightarrow\infty}\left(e^{a_{n}}-1-a_{n}\right)} & = & 0\\\\{\displaystyle \lim_{n\rightarrow\infty}\left(e^{b_{n}}-1-b_{n}\right)} & = & 0\\\\{\displaystyle \lim_{n\rightarrow\infty}\left(e^{c_{n}}-1-c_{n}\right)} & = & 0\end{array}\]

Montrons que ceci entraîne :

    \[\begin{array}{ccc}{\displaystyle \lim_{n\rightarrow\infty}a_{n}} & = & 0\\\\{\displaystyle \lim_{n\rightarrow\infty}b_{n}} & = & 0\\\\{\displaystyle \lim_{n\rightarrow\infty}c_{n}} & = & 0\end{array}\]


On observe que l’application

    \[ \varphi:\mathbb{R}\rightarrow\mathbb{R},\thinspace t\mapsto e^{t}-1-t\]


induit :

➭ une bijection décroissante u:\left]-\infty,0\right]\rightarrow\left[0,+\infty\right[,
➭ une bijection croissante v:\left[0,+\infty\right[\rightarrow\left[0,+\infty\right[.

Etant donné \epsilon>0, si t\in\mathbb{R} vérifie \varphi\left(t\right)\leqslant\min\left\{u\left(-\epsilon\right),\thinspace v\left(\epsilon\right)\right\} , alors de deux choses l’une :

  • ou bien t\leqslant0, auquel cas t\geqslant-\epsilon
  • ou bien t\geqslant0, auquel cas t\leqslant\epsilon

et par conséquent : \left|t\right|\leqslant\epsilon.

Comme {\displaystyle \lim_{n\rightarrow\infty}\left(e^{a_{n}}-1-a_n\right)=0}, il existe N\in\mathbb{N} tel que :

    \[\forall n\geqslant N,\,\varphi\left(a_{n}\right)\leqslant\min\left\{u\left(-\epsilon\right),v\left(\epsilon\right)\right\}\]

Ceci montre que : \forall n\geqslant N,\thinspace\left|a_{n}\right|\leqslant\epsilon

Ainsi : {\displaystyle \lim_{n\rightarrow\infty}a_{n}=0} et le même argument s’applique aux deux autres suites.


Si un point n’est pas clair ou vous paraît insuffisamment détaillé, n’hésitez pas à poster un commentaire ou à me joindre via le formulaire de contact.

Partager cet article

Laisser un commentaire